Multiple of Markov-chain The Next CEO of Stack OverflowExercise on Markov chainWhy a positive...

Reshaping json / reparing json inside shell script (remove trailing comma)

Help/tips for a first time writer?

Does the Idaho Potato Commission associate potato skins with healthy eating?

It is correct to match light sources with the same color temperature?

Why did early computer designers eschew integers?

Could a dragon use its wings to swim?

What is the process for cleansing a very negative action

TikZ: How to fill area with a special pattern?

Why don't programming languages automatically manage the synchronous/asynchronous problem?

Lucky Feat: How can "more than one creature spend a luck point to influence the outcome of a roll"?

Are the names of these months realistic?

Is it professional to write unrelated content in an almost-empty email?

Can I calculate next year's exemptions based on this year's refund/amount owed?

Is there a reasonable and studied concept of reduction between regular languages?

Getting Stale Gas Out of a Gas Tank w/out Dropping the Tank

What was Carter Burke's job for "the company" in Aliens?

IC has pull-down resistors on SMBus lines?

Why is information "lost" when it got into a black hole?

Expressing the idea of having a very busy time

Physiological effects of huge anime eyes

How to use ReplaceAll on an expression that contains a rule

What does "shotgun unity" refer to here in this sentence?

Is it ok to trim down a tube patch?

Help! I cannot understand this game’s notations!



Multiple of Markov-chain



The Next CEO of Stack OverflowExercise on Markov chainWhy a positive recurrent Markov chain implies positive limiting probability?Finite-state Markov chainIs any subchain of Markov chain also a Markov chain?Markov chain $Z_{n+1}=max(X_n+Z_n,0)$Irreducible aperiodic Markov chainGeneral Markov propertyWhat is the name of the following (Markov-like) chain?Finding Markov Chain of $Y_n = M_n - S_n$?Definition of Markov chain












0












$begingroup$


Let $(X_n)_{ninBbb N}$ be a Markov-chain, i.e.$$forall :ninBbb N:forall:a_1,ldots,a_n,a_{n+1}inBbb Z:Bbb P(X_{n+1}=a_{n+1}mid X_n=a_n,ldots,X_1=a_1)=Bbb P(X_{n+1}=a_{n+1}mid X_n=a_n)$$
Let $pinBbb N$. How can I proof that $(X_{pcdot n})_{ninBbb N}$ is a Markov-chain again?










share|cite|improve this question











$endgroup$

















    0












    $begingroup$


    Let $(X_n)_{ninBbb N}$ be a Markov-chain, i.e.$$forall :ninBbb N:forall:a_1,ldots,a_n,a_{n+1}inBbb Z:Bbb P(X_{n+1}=a_{n+1}mid X_n=a_n,ldots,X_1=a_1)=Bbb P(X_{n+1}=a_{n+1}mid X_n=a_n)$$
    Let $pinBbb N$. How can I proof that $(X_{pcdot n})_{ninBbb N}$ is a Markov-chain again?










    share|cite|improve this question











    $endgroup$















      0












      0








      0





      $begingroup$


      Let $(X_n)_{ninBbb N}$ be a Markov-chain, i.e.$$forall :ninBbb N:forall:a_1,ldots,a_n,a_{n+1}inBbb Z:Bbb P(X_{n+1}=a_{n+1}mid X_n=a_n,ldots,X_1=a_1)=Bbb P(X_{n+1}=a_{n+1}mid X_n=a_n)$$
      Let $pinBbb N$. How can I proof that $(X_{pcdot n})_{ninBbb N}$ is a Markov-chain again?










      share|cite|improve this question











      $endgroup$




      Let $(X_n)_{ninBbb N}$ be a Markov-chain, i.e.$$forall :ninBbb N:forall:a_1,ldots,a_n,a_{n+1}inBbb Z:Bbb P(X_{n+1}=a_{n+1}mid X_n=a_n,ldots,X_1=a_1)=Bbb P(X_{n+1}=a_{n+1}mid X_n=a_n)$$
      Let $pinBbb N$. How can I proof that $(X_{pcdot n})_{ninBbb N}$ is a Markov-chain again?







      probability probability-theory markov-chains






      share|cite|improve this question















      share|cite|improve this question













      share|cite|improve this question




      share|cite|improve this question








      edited Mar 17 at 17:00







      Daniel Banov

















      asked Mar 17 at 16:55









      Daniel BanovDaniel Banov

      217




      217






















          0






          active

          oldest

          votes












          Your Answer





          StackExchange.ifUsing("editor", function () {
          return StackExchange.using("mathjaxEditing", function () {
          StackExchange.MarkdownEditor.creationCallbacks.add(function (editor, postfix) {
          StackExchange.mathjaxEditing.prepareWmdForMathJax(editor, postfix, [["$", "$"], ["\\(","\\)"]]);
          });
          });
          }, "mathjax-editing");

          StackExchange.ready(function() {
          var channelOptions = {
          tags: "".split(" "),
          id: "69"
          };
          initTagRenderer("".split(" "), "".split(" "), channelOptions);

          StackExchange.using("externalEditor", function() {
          // Have to fire editor after snippets, if snippets enabled
          if (StackExchange.settings.snippets.snippetsEnabled) {
          StackExchange.using("snippets", function() {
          createEditor();
          });
          }
          else {
          createEditor();
          }
          });

          function createEditor() {
          StackExchange.prepareEditor({
          heartbeatType: 'answer',
          autoActivateHeartbeat: false,
          convertImagesToLinks: true,
          noModals: true,
          showLowRepImageUploadWarning: true,
          reputationToPostImages: 10,
          bindNavPrevention: true,
          postfix: "",
          imageUploader: {
          brandingHtml: "Powered by u003ca class="icon-imgur-white" href="https://imgur.com/"u003eu003c/au003e",
          contentPolicyHtml: "User contributions licensed under u003ca href="https://creativecommons.org/licenses/by-sa/3.0/"u003ecc by-sa 3.0 with attribution requiredu003c/au003e u003ca href="https://stackoverflow.com/legal/content-policy"u003e(content policy)u003c/au003e",
          allowUrls: true
          },
          noCode: true, onDemand: true,
          discardSelector: ".discard-answer"
          ,immediatelyShowMarkdownHelp:true
          });


          }
          });














          draft saved

          draft discarded


















          StackExchange.ready(
          function () {
          StackExchange.openid.initPostLogin('.new-post-login', 'https%3a%2f%2fmath.stackexchange.com%2fquestions%2f3151762%2fmultiple-of-markov-chain%23new-answer', 'question_page');
          }
          );

          Post as a guest















          Required, but never shown

























          0






          active

          oldest

          votes








          0






          active

          oldest

          votes









          active

          oldest

          votes






          active

          oldest

          votes
















          draft saved

          draft discarded




















































          Thanks for contributing an answer to Mathematics Stack Exchange!


          • Please be sure to answer the question. Provide details and share your research!

          But avoid



          • Asking for help, clarification, or responding to other answers.

          • Making statements based on opinion; back them up with references or personal experience.


          Use MathJax to format equations. MathJax reference.


          To learn more, see our tips on writing great answers.




          draft saved


          draft discarded














          StackExchange.ready(
          function () {
          StackExchange.openid.initPostLogin('.new-post-login', 'https%3a%2f%2fmath.stackexchange.com%2fquestions%2f3151762%2fmultiple-of-markov-chain%23new-answer', 'question_page');
          }
          );

          Post as a guest















          Required, but never shown





















































          Required, but never shown














          Required, but never shown












          Required, but never shown







          Required, but never shown

































          Required, but never shown














          Required, but never shown












          Required, but never shown







          Required, but never shown







          Popular posts from this blog

          Magento 2 - Add success message with knockout Planned maintenance scheduled April 23, 2019 at 23:30 UTC (7:30pm US/Eastern) Announcing the arrival of Valued Associate #679: Cesar Manara Unicorn Meta Zoo #1: Why another podcast?Success / Error message on ajax request$.widget is not a function when loading a homepage after add custom jQuery on custom themeHow can bind jQuery to current document in Magento 2 When template load by ajaxRedirect page using plugin in Magento 2Magento 2 - Update quantity and totals of cart page without page reload?Magento 2: Quote data not loaded on knockout checkoutMagento 2 : I need to change add to cart success message after adding product into cart through pluginMagento 2.2.5 How to add additional products to cart from new checkout step?Magento 2 Add error/success message with knockoutCan't validate Post Code on checkout page

          Fil:Tokke komm.svg

          Where did Arya get these scars? Unicorn Meta Zoo #1: Why another podcast? Announcing the arrival of Valued Associate #679: Cesar Manara Favourite questions and answers from the 1st quarter of 2019Why did Arya refuse to end it?Has the pronunciation of Arya Stark's name changed?Has Arya forgiven people?Why did Arya Stark lose her vision?Why can Arya still use the faces?Has the Narrow Sea become narrower?Does Arya Stark know how to make poisons outside of the House of Black and White?Why did Nymeria leave Arya?Why did Arya not kill the Lannister soldiers she encountered in the Riverlands?What is the current canonical age of Sansa, Bran and Arya Stark?